LSAT and Law School Admissions Forum

Get expert LSAT preparation and law school admissions advice from PowerScore Test Preparation.

 Administrator
PowerScore Staff
  • PowerScore Staff
  • Posts: 8917
  • Joined: Feb 02, 2011
|
#25629
Complete Question Explanation
(The complete setup for this game can be found here: lsat/viewtopic.php?t=3673)

The correct answer choice is (A)

Absent any Not-Law inferences, this Global, Cannot Be True question is likely to test your
understanding of the rules directly. As such, it should be approached by the process of
elimination.

Answer choice (A): This is the correct answer choice, because H cannot be the last film shown
on Friday.

Answer choice (B): This answer choice is incorrect, because L could be shown on each day of the
festival without violating any of the rules. If necessary, we can test this by creating the following
hypothetical:
june07_game_2_#7_diagram_1.png
Answer choice (C): This answer choice is incorrect, because G could be shown second on each
day of the festival without violating any of the rules. For instance:
june07_game_2_#7_diagram_2.png
Answer choice (D): This answer choice is also incorrect, because there are no rules governing the
selection of films that can be shown first on any of the days.

Answer choice (E): This answer choice is incorrect, because a different film could be shown last
on each day of the festival. For instance:
june07_game_2_#7_diagram_3.png
You do not have the required permissions to view the files attached to this post.
User avatar
 KwakuS
  • Posts: 35
  • Joined: Jun 03, 2021
|
#93787
Hello,

Thank you for this response. I see that process of elimination is used to solve this question. The template for this question has a lot of open-endedness, meaning that process of elimination could take a bit of time because one has to try each new solution. Is that still the best and most time-efficient approach?

Thanks,
Kwaku
User avatar
 Beth Hayden
PowerScore Staff
  • PowerScore Staff
  • Posts: 123
  • Joined: Sep 04, 2021
|
#93801
Hi Kwaku,

Actually, I didn't need to diagram out each answer choice to solve this question. The open-ended nature is a bit scary at first, but because there are only three variables and Friday and Saturday can only have two slots at most, there are actually not that many ways this game can go.

I knew A was correct immediately and didn't even need to parse the other answer choices, and here is why. On Friday, either G or L is the last film shown (because nothing can be after it). This means that it would be impossible for H to be last on Friday.

Hope that helps!
Beth
User avatar
 KwakuS
  • Posts: 35
  • Joined: Jun 03, 2021
|
#93814
Hello Beth,

Oh yeah, I see that now. Thank you!

Kwaku

Get the most out of your LSAT Prep Plus subscription.

Analyze and track your performance with our Testing and Analytics Package.